EOM de Heisenberg para ⟨x⟩⟨x⟩\langle x \rangle en estado propio de impulso: ¿dónde está mi error?

Ecuación de movimiento para el valor esperado de una partícula cuántica en un estado propio de momento:

d d t X = 1 i h [ X , H ]

y dado que está en un estado propio de impulso,

1 i h [ X , H ] = 1 i h pag | [ X , H ] | pag

Ampliando esto,

1 i h pag | ( X pag 2 2 metro + X V ( X ) ) ( pag 2 2 metro X + V ( X ) X ) | pag = 1 i h ( pag 2 2 metro pag 2 2 metro ) pag | X | pag + 1 i h pag | [ X , V ( X ) ] | pag = 0

desde V es V ( X ) . Pero d d t X = pag metro . ¿Dónde está mi error?

Tenga en cuenta que pag ^ es un operador, no puedes usarlo como una variable
Pista: que es pag | X | pag ?
Posibles duplicados: physics.stackexchange.com/q/14116/2451 y enlaces allí.

Respuestas (2)

Bueno, ya sabes que la función propia de pag ^ es Exp ( i pag X ) , así que tratemos de encontrar cuál es el valor esperado de X es, para empezar:

pag | X | pag = Exp ( i pag X ) X Exp ( i pag X ) d X = X d X
y esta integral no existe . Entonces no debería ser una sorpresa que tratar de tomar la derivada del tiempo no tenga sentido. La razón subyacente es que Exp ( i pag X ) no es normalizable. Esta respuesta a una pregunta similar brinda más detalles sobre cómo resolver esto.

¡gracias! esto dio en el clavo. <x'|p|x''>=ih delta(x'-x'')/(x'-x'') que aclara mi confusión (y es consistente con la forma del operador de impulso en la base de posición ya que delta(x)/x= - d delta(x)/dx

Actualización : Los comentarios de Robin señalaron mi confusión.

Considera lo siguiente:

[ X , pag 2 ] = X pag pag pag pag X = X pag pag pag X pag + pag X pag pag pag X = [ X , pag ] pag + pag [ X , pag ] = 2 i h pag

Si conecta esto en la expresión inicial, obtendrá exactamente lo que esperaría de este observable: pag / metro .

Pero las manipulaciones formalmente correctas que realizó proporcionan una respuesta diferente, mostrando que algo no está bien. Para la conclusión, vea la respuesta de Robin.


Respuesta original errónea:

No puede tomar el término de impulso fuera del promedio: el impulso p es un operador que no conmuta con x.

-1: Dado que el estado es un estado propio de pag ^ , puedes reemplazar pag ^ con el valor propio pag (actuando a la izquierda sobre el sujetador ya la derecha sobre el ket).
No, creo que no puedes. Siguiendo tu lógica, lo siguiente es cierto: X pag = X pag y por lo tanto [ X , pag ] = 0 .
Debe asegurarse de que todas las cantidades involucradas realmente existan, consulte aquí physics.stackexchange.com/q/14116/2451
Creo que tienes razón
Pero puedo obtener la respuesta correcta si uso la relación de conmutación de x y p, esto es confuso
@RobinEkman Actualicé mi respuesta para evitar más confusiones. Tengo una pregunta para usted: ¿tiene algún sentido considerar la evolución de la X ? Si evito evaluar esta expresión en el lado derecho, obtengo una respuesta físicamente buena. Pero tenemos al menos 2 formas de sacar tonterías (la tuya y la de la pregunta de arriba)